What is the minimum value of $|x| + |2x+1|+|3x+2|+cdots+|99x+98|$?












0












$begingroup$



What is the minimum value of the following?
$$A = |x| + |2x+1|+|3x+2|+cdots+|99x+98|$$




What I've tried so far:




  • Since $|x| = |-x| $ it is clear that $|3x + 2|$ = $|-3x - 2|$, $|5x + 4| = |-5x-4|$ and so on.

  • Therefore $A geq -x + 2x+1-3x-2+4x+3-5x-4+cdots-99x-98 = -50x - 49$ and I'm stuck here.


I'm quite sure it's not the right way to go, but that's what I've tried so far. Thanks in advance.










share|cite|improve this question











$endgroup$












  • $begingroup$
    @Yanko I'm not quite sure, might give it a try. Thanks for the suggestion
    $endgroup$
    – Brian Le
    Jan 13 at 14:25
















0












$begingroup$



What is the minimum value of the following?
$$A = |x| + |2x+1|+|3x+2|+cdots+|99x+98|$$




What I've tried so far:




  • Since $|x| = |-x| $ it is clear that $|3x + 2|$ = $|-3x - 2|$, $|5x + 4| = |-5x-4|$ and so on.

  • Therefore $A geq -x + 2x+1-3x-2+4x+3-5x-4+cdots-99x-98 = -50x - 49$ and I'm stuck here.


I'm quite sure it's not the right way to go, but that's what I've tried so far. Thanks in advance.










share|cite|improve this question











$endgroup$












  • $begingroup$
    @Yanko I'm not quite sure, might give it a try. Thanks for the suggestion
    $endgroup$
    – Brian Le
    Jan 13 at 14:25














0












0








0


1



$begingroup$



What is the minimum value of the following?
$$A = |x| + |2x+1|+|3x+2|+cdots+|99x+98|$$




What I've tried so far:




  • Since $|x| = |-x| $ it is clear that $|3x + 2|$ = $|-3x - 2|$, $|5x + 4| = |-5x-4|$ and so on.

  • Therefore $A geq -x + 2x+1-3x-2+4x+3-5x-4+cdots-99x-98 = -50x - 49$ and I'm stuck here.


I'm quite sure it's not the right way to go, but that's what I've tried so far. Thanks in advance.










share|cite|improve this question











$endgroup$





What is the minimum value of the following?
$$A = |x| + |2x+1|+|3x+2|+cdots+|99x+98|$$




What I've tried so far:




  • Since $|x| = |-x| $ it is clear that $|3x + 2|$ = $|-3x - 2|$, $|5x + 4| = |-5x-4|$ and so on.

  • Therefore $A geq -x + 2x+1-3x-2+4x+3-5x-4+cdots-99x-98 = -50x - 49$ and I'm stuck here.


I'm quite sure it's not the right way to go, but that's what I've tried so far. Thanks in advance.







absolute-value maxima-minima






share|cite|improve this question















share|cite|improve this question













share|cite|improve this question




share|cite|improve this question








edited Jan 13 at 14:26









Blue

48k870153




48k870153










asked Jan 13 at 14:18









Brian LeBrian Le

1033




1033












  • $begingroup$
    @Yanko I'm not quite sure, might give it a try. Thanks for the suggestion
    $endgroup$
    – Brian Le
    Jan 13 at 14:25


















  • $begingroup$
    @Yanko I'm not quite sure, might give it a try. Thanks for the suggestion
    $endgroup$
    – Brian Le
    Jan 13 at 14:25
















$begingroup$
@Yanko I'm not quite sure, might give it a try. Thanks for the suggestion
$endgroup$
– Brian Le
Jan 13 at 14:25




$begingroup$
@Yanko I'm not quite sure, might give it a try. Thanks for the suggestion
$endgroup$
– Brian Le
Jan 13 at 14:25










4 Answers
4






active

oldest

votes


















1












$begingroup$

The general term is of the form $f_n = |(n+1)x + n|$. For $x > -frac{n}{n+1} = c_n$ we have $f_n = (n+1)x + n$, for $x le -frac{n}{n+1}$ we have $f_n = -(n+1)x - n$. The terms $c_0 = 0 > c_n > c_{98} = -frac{98}{99}$.



So clearly $A$ will get large if $x>0$ or $x <-frac{98}{99}$.



So choose some $0 > x > -frac{98}{99}$. Then $c_{n-1} > x > c_n$ and
$$A = sum_{k=n}^{98} [(k+1)x + k] - sum_{k=0}^{n-1} [(k+1)x + k]\
= (98 - 2n)x + (frac12 98 cdot 99 - n (n+1))(x+1) \
= -(98 - 2n)frac{n}{n+1} + (49 cdot 99 - n (n+1))(-frac{n}{n+1} +1) \
=
frac{1}{n+1} left( -n (98 - 2n) + (49 cdot 99 - n (n+1))right) \
=
frac{n^2 - 99 n + 4851}{n+1}$$

and at $n =sqrt{4951} - 1 simeq 69.3$ this attains its local minimum $2 sqrt{4951} - 101 simeq 40 $.



So we should look at the neighbouring values to find the exact minimum of $A$ which will occur at some boundary.



For $n = 69$ and with the sum above we have that $A=frac{937}{23}simeq 40.739$.



For $n = 70$ we have that $A=frac{285}{7} simeq 40.714$ which is the lowest value, taken at $x = -frac{69}{70} simeq -0.9857$.






share|cite|improve this answer











$endgroup$













  • $begingroup$
    That's quite a lot of Maths. Do you think there's a simpler solution to this question?
    $endgroup$
    – Brian Le
    Jan 13 at 15:00










  • $begingroup$
    I don't know a simpler one.
    $endgroup$
    – Andreas
    Jan 13 at 15:33










  • $begingroup$
    Finally I got it. Thanks for the detailed answer :D
    $endgroup$
    – Brian Le
    Jan 13 at 19:37



















1












$begingroup$

Hint: try to solve it for the first two terms. Then add the third term. You should see a pattern emerging.






share|cite|improve this answer









$endgroup$













  • $begingroup$
    Hey, thanks for the answer! Was my initial method correct?
    $endgroup$
    – Brian Le
    Jan 13 at 14:43










  • $begingroup$
    You've only given a lower bound, and not an especially useful one: noting that $x > 0$ gives a stronger one.
    $endgroup$
    – user3482749
    Jan 13 at 14:45










  • $begingroup$
    Indeed. Try to make cases for the first terms and see when they are positive or negative before applying the absolute values.
    $endgroup$
    – Ashik4ga
    Jan 13 at 14:46



















1












$begingroup$

Since the function is piecewise linear, the minimum value, if it exists, must occur at one of the transition points (and it's straightforward to notice that for large values of $|x|$, the sum is large, so indeed the minimum value does occur).



Thus, the minimum value is achieved at $x = frac{1}{n} - 1$ for some $n in B = {1,ldots, 99}$.



Now, at some $x in (frac{1}{m+1} - 1,frac{1}{m} - 1)$ for some integer $m$ between $1$ and $98$ inclusive, the derivative of $A$ is given by



$$sumlimits_{i=m+1}^{99}i - sumlimits_{j=1}^{m}j = dfrac{(99-m)(100+m) - m(m+1)}{2} = 4950 - m - m^2,$$
which is $0$ when $m = frac{-1 pm sqrt{19801}}{2}$. One of the solutions is negative, the other is between 69 and 70. Thus, $A$ is decreasing on all of $(-infty, frac{1}{70}-1)$ and increasing on all of $(frac{1}{69}-1,infty)$, so our minimum is at either $x = frac{1}{69}-1$ or $x = frac{1}{70}-1$. Evaluating these two, we find that $$Aleft(frac{1}{69}-1right) = sumlimits_{i=70}^{99}left(frac{i}{69} -1right) + sumlimits_{j=1}^{69}left(1-frac{i}{69}right)=frac{937}{23}approx 40.739mbox{, and}$$
$$Aleft(frac{1}{70}-1right) = sumlimits_{i=71}^{99}left(frac{i}{70}-1right)+sumlimits_{j=1}^{70}left(1-frac{i}{70}right) = frac{285}{7} approx 40.714,$$



hence our minimum is at $x = frac{1}{70}-1$, with a value of $frac{285}{7}$.






share|cite|improve this answer









$endgroup$





















    1












    $begingroup$

    If a sequence $a_1le a_2le cdotsle a_N$ is given, it is known (and we can also prove) that
    $$
    sum_{i=1}^N|x-a_i|
    $$
    is minimized at the median of the sequence $(a_i)_{i=1}^N$. (Put differently, the least mean absolute deviation estimator is given by the median.) The median is defined to be $a_{frac{N+1}{2}}$ if $N$ is odd and is equal to any value in $[a_{frac{N}{2}},a_{frac{N}{2}+1}]$ if $N$ is even.



    In this case, by substituting $x=-z$, the objective function is
    $$
    sum_{i=1}^N|z-a_i|
    $$
    where ${a_i}={0,frac{1}{2},frac{1}{2},frac{2}{3},frac{2}{3},frac{2}{3},frac{3}{4},frac{3}{4},ldots,frac{98}{99}}$. Since $N=1+2+cdots +99 = 4950$, we should look for $2475$-th and $2476$-th term. Since there are $1+2+cdots +j=frac{j(j+1)}{2}$ terms in $${0,frac{1}{2},frac{1}{2},frac{2}{3},frac{2}{3},frac{2}{3},frac{3}{4},ldots,frac{j-1}{j},frac{j-1}{j}},$$ we know that there are $2485$ terms in
    $$
    {0,frac{1}{2},frac{1}{2},frac{2}{3},frac{2}{3},frac{2}{3},frac{3}{4},ldots,frac{69}{70},frac{69}{70}}.
    $$
    This shows that $2475$-th and $2476$-th terms are $frac{69}{70}$, and hence the minimum is attained by $x=-z=-frac{69}{70}$.






    share|cite|improve this answer











    $endgroup$













      Your Answer





      StackExchange.ifUsing("editor", function () {
      return StackExchange.using("mathjaxEditing", function () {
      StackExchange.MarkdownEditor.creationCallbacks.add(function (editor, postfix) {
      StackExchange.mathjaxEditing.prepareWmdForMathJax(editor, postfix, [["$", "$"], ["\\(","\\)"]]);
      });
      });
      }, "mathjax-editing");

      StackExchange.ready(function() {
      var channelOptions = {
      tags: "".split(" "),
      id: "69"
      };
      initTagRenderer("".split(" "), "".split(" "), channelOptions);

      StackExchange.using("externalEditor", function() {
      // Have to fire editor after snippets, if snippets enabled
      if (StackExchange.settings.snippets.snippetsEnabled) {
      StackExchange.using("snippets", function() {
      createEditor();
      });
      }
      else {
      createEditor();
      }
      });

      function createEditor() {
      StackExchange.prepareEditor({
      heartbeatType: 'answer',
      autoActivateHeartbeat: false,
      convertImagesToLinks: true,
      noModals: true,
      showLowRepImageUploadWarning: true,
      reputationToPostImages: 10,
      bindNavPrevention: true,
      postfix: "",
      imageUploader: {
      brandingHtml: "Powered by u003ca class="icon-imgur-white" href="https://imgur.com/"u003eu003c/au003e",
      contentPolicyHtml: "User contributions licensed under u003ca href="https://creativecommons.org/licenses/by-sa/3.0/"u003ecc by-sa 3.0 with attribution requiredu003c/au003e u003ca href="https://stackoverflow.com/legal/content-policy"u003e(content policy)u003c/au003e",
      allowUrls: true
      },
      noCode: true, onDemand: true,
      discardSelector: ".discard-answer"
      ,immediatelyShowMarkdownHelp:true
      });


      }
      });














      draft saved

      draft discarded


















      StackExchange.ready(
      function () {
      StackExchange.openid.initPostLogin('.new-post-login', 'https%3a%2f%2fmath.stackexchange.com%2fquestions%2f3072042%2fwhat-is-the-minimum-value-of-x-2x13x2-cdots99x98%23new-answer', 'question_page');
      }
      );

      Post as a guest















      Required, but never shown

























      4 Answers
      4






      active

      oldest

      votes








      4 Answers
      4






      active

      oldest

      votes









      active

      oldest

      votes






      active

      oldest

      votes









      1












      $begingroup$

      The general term is of the form $f_n = |(n+1)x + n|$. For $x > -frac{n}{n+1} = c_n$ we have $f_n = (n+1)x + n$, for $x le -frac{n}{n+1}$ we have $f_n = -(n+1)x - n$. The terms $c_0 = 0 > c_n > c_{98} = -frac{98}{99}$.



      So clearly $A$ will get large if $x>0$ or $x <-frac{98}{99}$.



      So choose some $0 > x > -frac{98}{99}$. Then $c_{n-1} > x > c_n$ and
      $$A = sum_{k=n}^{98} [(k+1)x + k] - sum_{k=0}^{n-1} [(k+1)x + k]\
      = (98 - 2n)x + (frac12 98 cdot 99 - n (n+1))(x+1) \
      = -(98 - 2n)frac{n}{n+1} + (49 cdot 99 - n (n+1))(-frac{n}{n+1} +1) \
      =
      frac{1}{n+1} left( -n (98 - 2n) + (49 cdot 99 - n (n+1))right) \
      =
      frac{n^2 - 99 n + 4851}{n+1}$$

      and at $n =sqrt{4951} - 1 simeq 69.3$ this attains its local minimum $2 sqrt{4951} - 101 simeq 40 $.



      So we should look at the neighbouring values to find the exact minimum of $A$ which will occur at some boundary.



      For $n = 69$ and with the sum above we have that $A=frac{937}{23}simeq 40.739$.



      For $n = 70$ we have that $A=frac{285}{7} simeq 40.714$ which is the lowest value, taken at $x = -frac{69}{70} simeq -0.9857$.






      share|cite|improve this answer











      $endgroup$













      • $begingroup$
        That's quite a lot of Maths. Do you think there's a simpler solution to this question?
        $endgroup$
        – Brian Le
        Jan 13 at 15:00










      • $begingroup$
        I don't know a simpler one.
        $endgroup$
        – Andreas
        Jan 13 at 15:33










      • $begingroup$
        Finally I got it. Thanks for the detailed answer :D
        $endgroup$
        – Brian Le
        Jan 13 at 19:37
















      1












      $begingroup$

      The general term is of the form $f_n = |(n+1)x + n|$. For $x > -frac{n}{n+1} = c_n$ we have $f_n = (n+1)x + n$, for $x le -frac{n}{n+1}$ we have $f_n = -(n+1)x - n$. The terms $c_0 = 0 > c_n > c_{98} = -frac{98}{99}$.



      So clearly $A$ will get large if $x>0$ or $x <-frac{98}{99}$.



      So choose some $0 > x > -frac{98}{99}$. Then $c_{n-1} > x > c_n$ and
      $$A = sum_{k=n}^{98} [(k+1)x + k] - sum_{k=0}^{n-1} [(k+1)x + k]\
      = (98 - 2n)x + (frac12 98 cdot 99 - n (n+1))(x+1) \
      = -(98 - 2n)frac{n}{n+1} + (49 cdot 99 - n (n+1))(-frac{n}{n+1} +1) \
      =
      frac{1}{n+1} left( -n (98 - 2n) + (49 cdot 99 - n (n+1))right) \
      =
      frac{n^2 - 99 n + 4851}{n+1}$$

      and at $n =sqrt{4951} - 1 simeq 69.3$ this attains its local minimum $2 sqrt{4951} - 101 simeq 40 $.



      So we should look at the neighbouring values to find the exact minimum of $A$ which will occur at some boundary.



      For $n = 69$ and with the sum above we have that $A=frac{937}{23}simeq 40.739$.



      For $n = 70$ we have that $A=frac{285}{7} simeq 40.714$ which is the lowest value, taken at $x = -frac{69}{70} simeq -0.9857$.






      share|cite|improve this answer











      $endgroup$













      • $begingroup$
        That's quite a lot of Maths. Do you think there's a simpler solution to this question?
        $endgroup$
        – Brian Le
        Jan 13 at 15:00










      • $begingroup$
        I don't know a simpler one.
        $endgroup$
        – Andreas
        Jan 13 at 15:33










      • $begingroup$
        Finally I got it. Thanks for the detailed answer :D
        $endgroup$
        – Brian Le
        Jan 13 at 19:37














      1












      1








      1





      $begingroup$

      The general term is of the form $f_n = |(n+1)x + n|$. For $x > -frac{n}{n+1} = c_n$ we have $f_n = (n+1)x + n$, for $x le -frac{n}{n+1}$ we have $f_n = -(n+1)x - n$. The terms $c_0 = 0 > c_n > c_{98} = -frac{98}{99}$.



      So clearly $A$ will get large if $x>0$ or $x <-frac{98}{99}$.



      So choose some $0 > x > -frac{98}{99}$. Then $c_{n-1} > x > c_n$ and
      $$A = sum_{k=n}^{98} [(k+1)x + k] - sum_{k=0}^{n-1} [(k+1)x + k]\
      = (98 - 2n)x + (frac12 98 cdot 99 - n (n+1))(x+1) \
      = -(98 - 2n)frac{n}{n+1} + (49 cdot 99 - n (n+1))(-frac{n}{n+1} +1) \
      =
      frac{1}{n+1} left( -n (98 - 2n) + (49 cdot 99 - n (n+1))right) \
      =
      frac{n^2 - 99 n + 4851}{n+1}$$

      and at $n =sqrt{4951} - 1 simeq 69.3$ this attains its local minimum $2 sqrt{4951} - 101 simeq 40 $.



      So we should look at the neighbouring values to find the exact minimum of $A$ which will occur at some boundary.



      For $n = 69$ and with the sum above we have that $A=frac{937}{23}simeq 40.739$.



      For $n = 70$ we have that $A=frac{285}{7} simeq 40.714$ which is the lowest value, taken at $x = -frac{69}{70} simeq -0.9857$.






      share|cite|improve this answer











      $endgroup$



      The general term is of the form $f_n = |(n+1)x + n|$. For $x > -frac{n}{n+1} = c_n$ we have $f_n = (n+1)x + n$, for $x le -frac{n}{n+1}$ we have $f_n = -(n+1)x - n$. The terms $c_0 = 0 > c_n > c_{98} = -frac{98}{99}$.



      So clearly $A$ will get large if $x>0$ or $x <-frac{98}{99}$.



      So choose some $0 > x > -frac{98}{99}$. Then $c_{n-1} > x > c_n$ and
      $$A = sum_{k=n}^{98} [(k+1)x + k] - sum_{k=0}^{n-1} [(k+1)x + k]\
      = (98 - 2n)x + (frac12 98 cdot 99 - n (n+1))(x+1) \
      = -(98 - 2n)frac{n}{n+1} + (49 cdot 99 - n (n+1))(-frac{n}{n+1} +1) \
      =
      frac{1}{n+1} left( -n (98 - 2n) + (49 cdot 99 - n (n+1))right) \
      =
      frac{n^2 - 99 n + 4851}{n+1}$$

      and at $n =sqrt{4951} - 1 simeq 69.3$ this attains its local minimum $2 sqrt{4951} - 101 simeq 40 $.



      So we should look at the neighbouring values to find the exact minimum of $A$ which will occur at some boundary.



      For $n = 69$ and with the sum above we have that $A=frac{937}{23}simeq 40.739$.



      For $n = 70$ we have that $A=frac{285}{7} simeq 40.714$ which is the lowest value, taken at $x = -frac{69}{70} simeq -0.9857$.







      share|cite|improve this answer














      share|cite|improve this answer



      share|cite|improve this answer








      edited Jan 13 at 15:33

























      answered Jan 13 at 14:54









      AndreasAndreas

      7,9801037




      7,9801037












      • $begingroup$
        That's quite a lot of Maths. Do you think there's a simpler solution to this question?
        $endgroup$
        – Brian Le
        Jan 13 at 15:00










      • $begingroup$
        I don't know a simpler one.
        $endgroup$
        – Andreas
        Jan 13 at 15:33










      • $begingroup$
        Finally I got it. Thanks for the detailed answer :D
        $endgroup$
        – Brian Le
        Jan 13 at 19:37


















      • $begingroup$
        That's quite a lot of Maths. Do you think there's a simpler solution to this question?
        $endgroup$
        – Brian Le
        Jan 13 at 15:00










      • $begingroup$
        I don't know a simpler one.
        $endgroup$
        – Andreas
        Jan 13 at 15:33










      • $begingroup$
        Finally I got it. Thanks for the detailed answer :D
        $endgroup$
        – Brian Le
        Jan 13 at 19:37
















      $begingroup$
      That's quite a lot of Maths. Do you think there's a simpler solution to this question?
      $endgroup$
      – Brian Le
      Jan 13 at 15:00




      $begingroup$
      That's quite a lot of Maths. Do you think there's a simpler solution to this question?
      $endgroup$
      – Brian Le
      Jan 13 at 15:00












      $begingroup$
      I don't know a simpler one.
      $endgroup$
      – Andreas
      Jan 13 at 15:33




      $begingroup$
      I don't know a simpler one.
      $endgroup$
      – Andreas
      Jan 13 at 15:33












      $begingroup$
      Finally I got it. Thanks for the detailed answer :D
      $endgroup$
      – Brian Le
      Jan 13 at 19:37




      $begingroup$
      Finally I got it. Thanks for the detailed answer :D
      $endgroup$
      – Brian Le
      Jan 13 at 19:37











      1












      $begingroup$

      Hint: try to solve it for the first two terms. Then add the third term. You should see a pattern emerging.






      share|cite|improve this answer









      $endgroup$













      • $begingroup$
        Hey, thanks for the answer! Was my initial method correct?
        $endgroup$
        – Brian Le
        Jan 13 at 14:43










      • $begingroup$
        You've only given a lower bound, and not an especially useful one: noting that $x > 0$ gives a stronger one.
        $endgroup$
        – user3482749
        Jan 13 at 14:45










      • $begingroup$
        Indeed. Try to make cases for the first terms and see when they are positive or negative before applying the absolute values.
        $endgroup$
        – Ashik4ga
        Jan 13 at 14:46
















      1












      $begingroup$

      Hint: try to solve it for the first two terms. Then add the third term. You should see a pattern emerging.






      share|cite|improve this answer









      $endgroup$













      • $begingroup$
        Hey, thanks for the answer! Was my initial method correct?
        $endgroup$
        – Brian Le
        Jan 13 at 14:43










      • $begingroup$
        You've only given a lower bound, and not an especially useful one: noting that $x > 0$ gives a stronger one.
        $endgroup$
        – user3482749
        Jan 13 at 14:45










      • $begingroup$
        Indeed. Try to make cases for the first terms and see when they are positive or negative before applying the absolute values.
        $endgroup$
        – Ashik4ga
        Jan 13 at 14:46














      1












      1








      1





      $begingroup$

      Hint: try to solve it for the first two terms. Then add the third term. You should see a pattern emerging.






      share|cite|improve this answer









      $endgroup$



      Hint: try to solve it for the first two terms. Then add the third term. You should see a pattern emerging.







      share|cite|improve this answer












      share|cite|improve this answer



      share|cite|improve this answer










      answered Jan 13 at 14:42









      Ashik4gaAshik4ga

      363




      363












      • $begingroup$
        Hey, thanks for the answer! Was my initial method correct?
        $endgroup$
        – Brian Le
        Jan 13 at 14:43










      • $begingroup$
        You've only given a lower bound, and not an especially useful one: noting that $x > 0$ gives a stronger one.
        $endgroup$
        – user3482749
        Jan 13 at 14:45










      • $begingroup$
        Indeed. Try to make cases for the first terms and see when they are positive or negative before applying the absolute values.
        $endgroup$
        – Ashik4ga
        Jan 13 at 14:46


















      • $begingroup$
        Hey, thanks for the answer! Was my initial method correct?
        $endgroup$
        – Brian Le
        Jan 13 at 14:43










      • $begingroup$
        You've only given a lower bound, and not an especially useful one: noting that $x > 0$ gives a stronger one.
        $endgroup$
        – user3482749
        Jan 13 at 14:45










      • $begingroup$
        Indeed. Try to make cases for the first terms and see when they are positive or negative before applying the absolute values.
        $endgroup$
        – Ashik4ga
        Jan 13 at 14:46
















      $begingroup$
      Hey, thanks for the answer! Was my initial method correct?
      $endgroup$
      – Brian Le
      Jan 13 at 14:43




      $begingroup$
      Hey, thanks for the answer! Was my initial method correct?
      $endgroup$
      – Brian Le
      Jan 13 at 14:43












      $begingroup$
      You've only given a lower bound, and not an especially useful one: noting that $x > 0$ gives a stronger one.
      $endgroup$
      – user3482749
      Jan 13 at 14:45




      $begingroup$
      You've only given a lower bound, and not an especially useful one: noting that $x > 0$ gives a stronger one.
      $endgroup$
      – user3482749
      Jan 13 at 14:45












      $begingroup$
      Indeed. Try to make cases for the first terms and see when they are positive or negative before applying the absolute values.
      $endgroup$
      – Ashik4ga
      Jan 13 at 14:46




      $begingroup$
      Indeed. Try to make cases for the first terms and see when they are positive or negative before applying the absolute values.
      $endgroup$
      – Ashik4ga
      Jan 13 at 14:46











      1












      $begingroup$

      Since the function is piecewise linear, the minimum value, if it exists, must occur at one of the transition points (and it's straightforward to notice that for large values of $|x|$, the sum is large, so indeed the minimum value does occur).



      Thus, the minimum value is achieved at $x = frac{1}{n} - 1$ for some $n in B = {1,ldots, 99}$.



      Now, at some $x in (frac{1}{m+1} - 1,frac{1}{m} - 1)$ for some integer $m$ between $1$ and $98$ inclusive, the derivative of $A$ is given by



      $$sumlimits_{i=m+1}^{99}i - sumlimits_{j=1}^{m}j = dfrac{(99-m)(100+m) - m(m+1)}{2} = 4950 - m - m^2,$$
      which is $0$ when $m = frac{-1 pm sqrt{19801}}{2}$. One of the solutions is negative, the other is between 69 and 70. Thus, $A$ is decreasing on all of $(-infty, frac{1}{70}-1)$ and increasing on all of $(frac{1}{69}-1,infty)$, so our minimum is at either $x = frac{1}{69}-1$ or $x = frac{1}{70}-1$. Evaluating these two, we find that $$Aleft(frac{1}{69}-1right) = sumlimits_{i=70}^{99}left(frac{i}{69} -1right) + sumlimits_{j=1}^{69}left(1-frac{i}{69}right)=frac{937}{23}approx 40.739mbox{, and}$$
      $$Aleft(frac{1}{70}-1right) = sumlimits_{i=71}^{99}left(frac{i}{70}-1right)+sumlimits_{j=1}^{70}left(1-frac{i}{70}right) = frac{285}{7} approx 40.714,$$



      hence our minimum is at $x = frac{1}{70}-1$, with a value of $frac{285}{7}$.






      share|cite|improve this answer









      $endgroup$


















        1












        $begingroup$

        Since the function is piecewise linear, the minimum value, if it exists, must occur at one of the transition points (and it's straightforward to notice that for large values of $|x|$, the sum is large, so indeed the minimum value does occur).



        Thus, the minimum value is achieved at $x = frac{1}{n} - 1$ for some $n in B = {1,ldots, 99}$.



        Now, at some $x in (frac{1}{m+1} - 1,frac{1}{m} - 1)$ for some integer $m$ between $1$ and $98$ inclusive, the derivative of $A$ is given by



        $$sumlimits_{i=m+1}^{99}i - sumlimits_{j=1}^{m}j = dfrac{(99-m)(100+m) - m(m+1)}{2} = 4950 - m - m^2,$$
        which is $0$ when $m = frac{-1 pm sqrt{19801}}{2}$. One of the solutions is negative, the other is between 69 and 70. Thus, $A$ is decreasing on all of $(-infty, frac{1}{70}-1)$ and increasing on all of $(frac{1}{69}-1,infty)$, so our minimum is at either $x = frac{1}{69}-1$ or $x = frac{1}{70}-1$. Evaluating these two, we find that $$Aleft(frac{1}{69}-1right) = sumlimits_{i=70}^{99}left(frac{i}{69} -1right) + sumlimits_{j=1}^{69}left(1-frac{i}{69}right)=frac{937}{23}approx 40.739mbox{, and}$$
        $$Aleft(frac{1}{70}-1right) = sumlimits_{i=71}^{99}left(frac{i}{70}-1right)+sumlimits_{j=1}^{70}left(1-frac{i}{70}right) = frac{285}{7} approx 40.714,$$



        hence our minimum is at $x = frac{1}{70}-1$, with a value of $frac{285}{7}$.






        share|cite|improve this answer









        $endgroup$
















          1












          1








          1





          $begingroup$

          Since the function is piecewise linear, the minimum value, if it exists, must occur at one of the transition points (and it's straightforward to notice that for large values of $|x|$, the sum is large, so indeed the minimum value does occur).



          Thus, the minimum value is achieved at $x = frac{1}{n} - 1$ for some $n in B = {1,ldots, 99}$.



          Now, at some $x in (frac{1}{m+1} - 1,frac{1}{m} - 1)$ for some integer $m$ between $1$ and $98$ inclusive, the derivative of $A$ is given by



          $$sumlimits_{i=m+1}^{99}i - sumlimits_{j=1}^{m}j = dfrac{(99-m)(100+m) - m(m+1)}{2} = 4950 - m - m^2,$$
          which is $0$ when $m = frac{-1 pm sqrt{19801}}{2}$. One of the solutions is negative, the other is between 69 and 70. Thus, $A$ is decreasing on all of $(-infty, frac{1}{70}-1)$ and increasing on all of $(frac{1}{69}-1,infty)$, so our minimum is at either $x = frac{1}{69}-1$ or $x = frac{1}{70}-1$. Evaluating these two, we find that $$Aleft(frac{1}{69}-1right) = sumlimits_{i=70}^{99}left(frac{i}{69} -1right) + sumlimits_{j=1}^{69}left(1-frac{i}{69}right)=frac{937}{23}approx 40.739mbox{, and}$$
          $$Aleft(frac{1}{70}-1right) = sumlimits_{i=71}^{99}left(frac{i}{70}-1right)+sumlimits_{j=1}^{70}left(1-frac{i}{70}right) = frac{285}{7} approx 40.714,$$



          hence our minimum is at $x = frac{1}{70}-1$, with a value of $frac{285}{7}$.






          share|cite|improve this answer









          $endgroup$



          Since the function is piecewise linear, the minimum value, if it exists, must occur at one of the transition points (and it's straightforward to notice that for large values of $|x|$, the sum is large, so indeed the minimum value does occur).



          Thus, the minimum value is achieved at $x = frac{1}{n} - 1$ for some $n in B = {1,ldots, 99}$.



          Now, at some $x in (frac{1}{m+1} - 1,frac{1}{m} - 1)$ for some integer $m$ between $1$ and $98$ inclusive, the derivative of $A$ is given by



          $$sumlimits_{i=m+1}^{99}i - sumlimits_{j=1}^{m}j = dfrac{(99-m)(100+m) - m(m+1)}{2} = 4950 - m - m^2,$$
          which is $0$ when $m = frac{-1 pm sqrt{19801}}{2}$. One of the solutions is negative, the other is between 69 and 70. Thus, $A$ is decreasing on all of $(-infty, frac{1}{70}-1)$ and increasing on all of $(frac{1}{69}-1,infty)$, so our minimum is at either $x = frac{1}{69}-1$ or $x = frac{1}{70}-1$. Evaluating these two, we find that $$Aleft(frac{1}{69}-1right) = sumlimits_{i=70}^{99}left(frac{i}{69} -1right) + sumlimits_{j=1}^{69}left(1-frac{i}{69}right)=frac{937}{23}approx 40.739mbox{, and}$$
          $$Aleft(frac{1}{70}-1right) = sumlimits_{i=71}^{99}left(frac{i}{70}-1right)+sumlimits_{j=1}^{70}left(1-frac{i}{70}right) = frac{285}{7} approx 40.714,$$



          hence our minimum is at $x = frac{1}{70}-1$, with a value of $frac{285}{7}$.







          share|cite|improve this answer












          share|cite|improve this answer



          share|cite|improve this answer










          answered Jan 13 at 15:24









          user3482749user3482749

          4,206919




          4,206919























              1












              $begingroup$

              If a sequence $a_1le a_2le cdotsle a_N$ is given, it is known (and we can also prove) that
              $$
              sum_{i=1}^N|x-a_i|
              $$
              is minimized at the median of the sequence $(a_i)_{i=1}^N$. (Put differently, the least mean absolute deviation estimator is given by the median.) The median is defined to be $a_{frac{N+1}{2}}$ if $N$ is odd and is equal to any value in $[a_{frac{N}{2}},a_{frac{N}{2}+1}]$ if $N$ is even.



              In this case, by substituting $x=-z$, the objective function is
              $$
              sum_{i=1}^N|z-a_i|
              $$
              where ${a_i}={0,frac{1}{2},frac{1}{2},frac{2}{3},frac{2}{3},frac{2}{3},frac{3}{4},frac{3}{4},ldots,frac{98}{99}}$. Since $N=1+2+cdots +99 = 4950$, we should look for $2475$-th and $2476$-th term. Since there are $1+2+cdots +j=frac{j(j+1)}{2}$ terms in $${0,frac{1}{2},frac{1}{2},frac{2}{3},frac{2}{3},frac{2}{3},frac{3}{4},ldots,frac{j-1}{j},frac{j-1}{j}},$$ we know that there are $2485$ terms in
              $$
              {0,frac{1}{2},frac{1}{2},frac{2}{3},frac{2}{3},frac{2}{3},frac{3}{4},ldots,frac{69}{70},frac{69}{70}}.
              $$
              This shows that $2475$-th and $2476$-th terms are $frac{69}{70}$, and hence the minimum is attained by $x=-z=-frac{69}{70}$.






              share|cite|improve this answer











              $endgroup$


















                1












                $begingroup$

                If a sequence $a_1le a_2le cdotsle a_N$ is given, it is known (and we can also prove) that
                $$
                sum_{i=1}^N|x-a_i|
                $$
                is minimized at the median of the sequence $(a_i)_{i=1}^N$. (Put differently, the least mean absolute deviation estimator is given by the median.) The median is defined to be $a_{frac{N+1}{2}}$ if $N$ is odd and is equal to any value in $[a_{frac{N}{2}},a_{frac{N}{2}+1}]$ if $N$ is even.



                In this case, by substituting $x=-z$, the objective function is
                $$
                sum_{i=1}^N|z-a_i|
                $$
                where ${a_i}={0,frac{1}{2},frac{1}{2},frac{2}{3},frac{2}{3},frac{2}{3},frac{3}{4},frac{3}{4},ldots,frac{98}{99}}$. Since $N=1+2+cdots +99 = 4950$, we should look for $2475$-th and $2476$-th term. Since there are $1+2+cdots +j=frac{j(j+1)}{2}$ terms in $${0,frac{1}{2},frac{1}{2},frac{2}{3},frac{2}{3},frac{2}{3},frac{3}{4},ldots,frac{j-1}{j},frac{j-1}{j}},$$ we know that there are $2485$ terms in
                $$
                {0,frac{1}{2},frac{1}{2},frac{2}{3},frac{2}{3},frac{2}{3},frac{3}{4},ldots,frac{69}{70},frac{69}{70}}.
                $$
                This shows that $2475$-th and $2476$-th terms are $frac{69}{70}$, and hence the minimum is attained by $x=-z=-frac{69}{70}$.






                share|cite|improve this answer











                $endgroup$
















                  1












                  1








                  1





                  $begingroup$

                  If a sequence $a_1le a_2le cdotsle a_N$ is given, it is known (and we can also prove) that
                  $$
                  sum_{i=1}^N|x-a_i|
                  $$
                  is minimized at the median of the sequence $(a_i)_{i=1}^N$. (Put differently, the least mean absolute deviation estimator is given by the median.) The median is defined to be $a_{frac{N+1}{2}}$ if $N$ is odd and is equal to any value in $[a_{frac{N}{2}},a_{frac{N}{2}+1}]$ if $N$ is even.



                  In this case, by substituting $x=-z$, the objective function is
                  $$
                  sum_{i=1}^N|z-a_i|
                  $$
                  where ${a_i}={0,frac{1}{2},frac{1}{2},frac{2}{3},frac{2}{3},frac{2}{3},frac{3}{4},frac{3}{4},ldots,frac{98}{99}}$. Since $N=1+2+cdots +99 = 4950$, we should look for $2475$-th and $2476$-th term. Since there are $1+2+cdots +j=frac{j(j+1)}{2}$ terms in $${0,frac{1}{2},frac{1}{2},frac{2}{3},frac{2}{3},frac{2}{3},frac{3}{4},ldots,frac{j-1}{j},frac{j-1}{j}},$$ we know that there are $2485$ terms in
                  $$
                  {0,frac{1}{2},frac{1}{2},frac{2}{3},frac{2}{3},frac{2}{3},frac{3}{4},ldots,frac{69}{70},frac{69}{70}}.
                  $$
                  This shows that $2475$-th and $2476$-th terms are $frac{69}{70}$, and hence the minimum is attained by $x=-z=-frac{69}{70}$.






                  share|cite|improve this answer











                  $endgroup$



                  If a sequence $a_1le a_2le cdotsle a_N$ is given, it is known (and we can also prove) that
                  $$
                  sum_{i=1}^N|x-a_i|
                  $$
                  is minimized at the median of the sequence $(a_i)_{i=1}^N$. (Put differently, the least mean absolute deviation estimator is given by the median.) The median is defined to be $a_{frac{N+1}{2}}$ if $N$ is odd and is equal to any value in $[a_{frac{N}{2}},a_{frac{N}{2}+1}]$ if $N$ is even.



                  In this case, by substituting $x=-z$, the objective function is
                  $$
                  sum_{i=1}^N|z-a_i|
                  $$
                  where ${a_i}={0,frac{1}{2},frac{1}{2},frac{2}{3},frac{2}{3},frac{2}{3},frac{3}{4},frac{3}{4},ldots,frac{98}{99}}$. Since $N=1+2+cdots +99 = 4950$, we should look for $2475$-th and $2476$-th term. Since there are $1+2+cdots +j=frac{j(j+1)}{2}$ terms in $${0,frac{1}{2},frac{1}{2},frac{2}{3},frac{2}{3},frac{2}{3},frac{3}{4},ldots,frac{j-1}{j},frac{j-1}{j}},$$ we know that there are $2485$ terms in
                  $$
                  {0,frac{1}{2},frac{1}{2},frac{2}{3},frac{2}{3},frac{2}{3},frac{3}{4},ldots,frac{69}{70},frac{69}{70}}.
                  $$
                  This shows that $2475$-th and $2476$-th terms are $frac{69}{70}$, and hence the minimum is attained by $x=-z=-frac{69}{70}$.







                  share|cite|improve this answer














                  share|cite|improve this answer



                  share|cite|improve this answer








                  edited Jan 13 at 15:30

























                  answered Jan 13 at 15:11









                  SongSong

                  11.1k628




                  11.1k628






























                      draft saved

                      draft discarded




















































                      Thanks for contributing an answer to Mathematics Stack Exchange!


                      • Please be sure to answer the question. Provide details and share your research!

                      But avoid



                      • Asking for help, clarification, or responding to other answers.

                      • Making statements based on opinion; back them up with references or personal experience.


                      Use MathJax to format equations. MathJax reference.


                      To learn more, see our tips on writing great answers.




                      draft saved


                      draft discarded














                      StackExchange.ready(
                      function () {
                      StackExchange.openid.initPostLogin('.new-post-login', 'https%3a%2f%2fmath.stackexchange.com%2fquestions%2f3072042%2fwhat-is-the-minimum-value-of-x-2x13x2-cdots99x98%23new-answer', 'question_page');
                      }
                      );

                      Post as a guest















                      Required, but never shown





















































                      Required, but never shown














                      Required, but never shown












                      Required, but never shown







                      Required, but never shown

































                      Required, but never shown














                      Required, but never shown












                      Required, but never shown







                      Required, but never shown







                      Popular posts from this blog

                      Mario Kart Wii

                      What does “Dominus providebit” mean?

                      Antonio Litta Visconti Arese